Initialisation

Tony Schwarzer
Modifié (October 2022) dans Algèbre
Bonjour,
Cela paraît simple mais je ne vois pas comment prouver l'initialisation de la question 17a)
En effet, soit $\lambda$ tel que $\lambda \sum_{i=1}^n t_i = 1$ (donc $\frac{k+1}{k}>\lambda >1$). J'ai essayé d'appliquer la question 16 avec les $\lambda t_i \geq 0$ et $N=k$, mais cela donne : $kp_i<1+qt_i(k+1)$, ce qui n'est pas le résultat voulu bien que pas loin...

Réponses

  • GaBuZoMeu
    Modifié (October 2022)
    Bonsoir
    Voyons, déroulons les choses pour $n=1$ en prenant comme indiqué $\alpha(k,1)=\dfrac1{k+1}$.
    Il s'agit donc de montrer que pour tout $t$ avec $1>t>\dfrac{k}{k+1}$, il existe un entier positif ou nul $p$ tel que $p>0$ et que $(kp+1)t > kp$.
    Ça ne me semble pas trop dur.
  • C’est sûr que si je ne fais pas attention et que j’oublie que n=1 c’est plus dur. Merci.
  • Pour la 17)c je ne vois pas trop d'où sort l'inégalité de droite. Une indication ?



  • Bonjour,

    d'où vient le sujet @Tony Schwarzer ?
  • C'est un complément de sujet de l'X qui démontre les résultats admis du sujet. (maths A 2016)
  • Mais déjà on est d'accord que pour la 17)c il convient d'appliquer la 16 avec $N$ défini en début de question 17a) ?
    J'ai beau chercher je ne vois pas d'où l'inégalité de droite sort.
  • Tony Schwarzer
    Modifié (November 2022)
    Je me permets d'actualiser cette page. Je ne vois toujours pas d'où sort l'inégalité de droite de la 17)c). J'ai essayé plusieurs choses, en particulier j'ai essayé d'appliquer (iv) de 16 avec $t_n'$. Cela ne donne rien.
Connectez-vous ou Inscrivez-vous pour répondre.